LSAT and Law School Admissions Forum

Get expert LSAT preparation and law school admissions advice from PowerScore Test Preparation.

 Administrator
PowerScore Staff
  • PowerScore Staff
  • Posts: 8917
  • Joined: Feb 02, 2011
|
#25630
Complete Question Explanation
(The complete setup for this game can be found here: lsat/viewtopic.php?t=3673)

The correct answer choice is (C)

To attack this question, use proper List question technique: take one rule and apply it to all
answer choices, then take another rule, and apply it to the remaining answer choices, and so on,
until only one answer choice remains. Due to the heavy Numerical element in this game, make
sure to also check the numerical restrictions established by the scenario in order to eliminate
pesky contenders for violating these restrictions (i.e. make sure that at least one film is shown on
each day, and also that all three films are shown during the festival).

Answer choice (A): This answer choice violates the requirement in the second sentence of the
scenario, namely, that each film be shown at least once during the festival. Since G is not shown
on any day, this answer choice is incorrect.

Answer choice (B): This answer choice violates the second rule and is therefore incorrect.

Answer choice (C): This is the correct answer choice.

Answer choice (D): This answer choice violates the first rule and is therefore incorrect.

Answer choice (E): This answer choice violates the second rule and is therefore incorrect.
Surprisingly, none of the four incorrect answer choices violates the third rule.
 Morgan O'Donnell
PowerScore Staff
  • PowerScore Staff
  • Posts: 67
  • Joined: Jun 25, 2012
|
#6227
One more...

June 2007, Game #2, Question #6
 Jon Denning
PowerScore Staff
  • PowerScore Staff
  • Posts: 904
  • Joined: Apr 11, 2011
|
#6230
Thanks for the question! This one is a little tricky, because there is a fair amount of uncertainty in exactly how many films will be shown each day, and exactly what their order must be. However with only three rules governing the placements the overall setup is still fairly manageable.

Basically there is one rule for each day, and that rule specifies which film(s) will be shown last. On Thursday H is last, on Friday either G or L is last (and the other can't be shown), and on Saturday either G or H is last (and the other can't be shown). That establishes some minimums and maximums numerically: 3 as min (would be H, L, G on Thurs, Fri, Sat), and 7 as max (3 Thurs, 2 Fri, 2 Sat).

Now just use those rules and the numerical restrictions they create to attack the questions.

For question 6, simply pick a rule and find a violation of it to eliminate four answers:

All three films are shown: eliminates A (no G)
H is the last film on Thursday: eliminates D (L is last)
G or L last on Friday, not both shown: eliminates B (both shown) and E (H last on Friday)

So C is correct.

Hope that helps!

Jon

Get the most out of your LSAT Prep Plus subscription.

Analyze and track your performance with our Testing and Analytics Package.